Difference between revisions of "2003 AMC 10A Problems/Problem 2"

m
(Redirected page to 2003 AMC 12A Problems/Problem 2)
 
Line 1: Line 1:
== Problem ==
+
#REDIRECT[[2003 AMC 12A Problems/Problem 2]]
Members of the Rockham Soccer League buy socks and T-shirts. Socks cost $4 per pair and each T-shirt costs $5 more than a pair of socks. Each member needs one pair of socks and a shirt for home games and another pair of socks and a shirt for away games. If the total cost is $2366, how many members are in the League?
 
 
 
<math> \mathrm{(A) \ } 77\qquad \mathrm{(B) \ } 91\qquad \mathrm{(C) \ } 143\qquad \mathrm{(D) \ } 182\qquad \mathrm{(E) \ } 286 </math>
 
 
 
== Solution ==
 
Since T-shirts cost <math>5</math> dollars more than a pair of socks, T-shirts cost <math>5+4=9</math> dollars.
 
 
 
Since each member needs <math>2</math> pairs of socks and <math>2</math> T-shirts, the total cost for <math>1</math> member is <math>2(4+9)=26</math> dollars.
 
 
 
Since <math>2366</math> dollars was the cost for the club, and <math>26</math> was the cost per member, the number of members in the League is <math>\frac{2366}{26}=91 \Rightarrow B</math>.
 
 
 
== See also ==
 
{{AMC10 box|year=2003|ab=A|num-b=1|num-a=3}}
 
 
 
[[Category:Introductory Algebra Problems]]
 

Latest revision as of 16:55, 29 July 2011